Toán Bất đẳng thức

B

bosjeunhan

Cấy ny em chế từ thất bại, ai hay vô hm và quan tâm tới BĐT nhì sẽ biết chế từ chỗ mô(Nhất là anh bboy114crew)
Có thể nó có thể gặp ở đâu đó rồi, nhưng thật sự em chế ra nó và có cách giải của riêng mình, mọi người coi thử:

Bài 26 : Cho các số thực dương thỏa mãn $a+b+c=3$ chứng minh rằng:
[TEX]\frac{a}{a+1} + \frac{b}{b+1} + \frac{c}{c+1} \leq \frac{3}{2} [/TEX]
 
Last edited by a moderator:
B

bboy114crew

Cấy ny em chế từ thất bại, ai hay vô hm và quan tâm tới BĐT nhì sẽ biết chế từ chỗ mô(Nhất là anh bboy114crew)
Có thể nó có thể gặp ở đâu đó rồi, nhưng thật sự em chế ra nó và có cách giải của riêng mình, mọi người coi thử:

Bài : Cho các số thực dương thỏa mãn $a+b+c=3$ chứng minh rằng:
[TEX]\frac{a}{a+1} + \frac{b}{b+1} + \frac{c}{c+1} \leq \frac{3}{2} [/TEX]

Bài này ta chỉ cần làm như sau!
BĐT cần chứng minh tương đương với:
[TEX]\sum(1- \frac{a}{a+1}) \geq 3- \frac{3}{2}[/TEX]
[TEX]\Leftrightarrow \sum \frac{1}{a+1} \geq \frac{3}{2}[/TEX]
BĐT này hiển nhiên đúng theo C-S!
Về cái motip sử dụng cái kĩ thuật anh vừa dùng còn nhiều bài lắm!
p\s: Ý em nói là sao anh không hiểu!
 
B

bosjeunhan

Bài này ta chỉ cần làm như sau!
BĐT cần chứng minh tương đương với:
[TEX]\sum(1- \frac{a}{a+1}) \geq 3- \frac{3}{2}[/TEX]
[TEX]\Leftrightarrow \sum \frac{1}{a+1} \geq \frac{3}{2}[/TEX]
BĐT này hiển nhiên đúng theo C-S!
Về cái motip sử dụng cái kĩ thuật anh vừa dùng còn nhiều bài lắm!
p\s: Ý em nói là sao anh không hiểu!

Anh hok thấy là em chế từ cấy bài em hỏi anh à Đây nỳ.

Cauchy cấy cục nớ xong đảo lên thôi anh ợ. Cách của em là như anh đó:

Cho $a+b+c=3$ (số thực dương)
CMR:

[TEX]\frac{a}{a+\sqrt[]{3a+bc}} + \frac{b}{b+\sqrt[]{3b+ac}} + \frac{c}{c+\sqrt[]{3c+ab}} \leq 1 [/TEX]
Bài này thế này nhé!
Ta có:
[TEX]\sum \frac{a}{a+\sqrt{3a+bc}}[/TEX]
[TEX]= \sum \frac{a}{a+\sqrt{(a+b)(a+c)}}[/TEX]
[TEX] \leq \sum \frac{a}{2a+\sqrt{bc}}[/TEX]
Ta chỉ cần chứng minh:
[TEX]\sum \frac{a}{2a+\sqrt{bc}} \leq 1[/TEX]
[TEX]\Leftrightarrow \sum \frac{\sqrt{bc}}{2a+\sqrt{bc}} \geq 1[/TEX]
Điều này luôn đúng theo Cauchy -Schwarz !;)
Một bài nữa, ấy mà em toàn chế bài có liên quan tới anh nák:

Bài 27:
Cho 3 số thực dương thỏa mãn [TEX]3abc \geq ab+bc+ca[/TEX] CMR:
[TEX]\frac{1}{\sqrt[]{a}}+\frac{1}{\sqrt[]{b}}+\frac{1}{\sqrt[]{c}} \geq \frac{9}{a^2+b^2+c^2}[/TEX]

Anh có thấy quen không ạ ;))


Viết cấy đề lại không người khác lại nhầm, rồi chứng minh mô mô nớ vs em cụng ghi sai dấu dẫn đến hiểu lầm.

P/s: Em del mấy cây spam đi nhá
 
Last edited by a moderator:
V

vy000

Một bài nữa, ấy mà em toàn chế bài có liên quan tới anh nák:
Cho 3 số thực dương thỏa mãn $3abc=ab+bc+ca$ CMR:
[TEX]\sum \frac{1}{\sqrt[]{a}} \geq \frac{9}{a^2+b^2+c^2}[/TEX]

Anh có thấy quen không ạ ;))
[/FONT][/SIZE]

\Rightarrow [TEX]\frac{1}{a}+\frac{1}{b}+\frac{1}{c}=3[/TEX]
\Rightarrow a+b+c\geq3 \Rightarrow [TEX]a^2+b^2+c^2\geq3[/TEX]
\Rightarrow [TEX](a^2+b^2+c^2)^2\geq(a+b+c)^2\geq3(a+b+c)\geq(\sqrt[]{a}+\sqrt[]{b}+\sqrt[]{c})^2[/TEX]
\Rightarrow [TEX]\frac{1}{(\sqrt[]{a}+\sqrt[]{b}+\sqrt[]{c})}\geq\frac{1}{a^2+b^2+c^2}[/TEX]

Coi lại cái đề dùm cái
 
Last edited by a moderator:
B

bosjeunhan

\Rightarrow [TEX]\frac{1}{a}+\frac{1}{b}+\frac{1}{c}=3[/TEX]
\Rightarrow a+b+c\geq3 \Rightarrow [TEX]a^2+b^2+c^2\geq3[/TEX]
\Rightarrow [TEX](a^2+b^2+c^2)^2\geq(a+b+c)^2\geq3(a+b+c)\geq(\sqrt[]{a}+\sqrt[]{b}+\sqrt[]{c})^2[/TEX]
\Rightarrow [TEX]\frac{1}{(\sqrt[]{a}+\sqrt[]{b}+\sqrt[]{c})}\geq\frac{1}{a^2+b^2+c^2}[/TEX]

Nếu đề như hồi đêm thì BĐT bạn nghĩ ra và tự chứng minh là đúng ;)), Cảm ơn bạn.

Nhưng, đề nghị bạn xem lại đề bài, mình bổ sung những dữ kiện cần thiết. Tất cả mọi người ak !
 
Last edited by a moderator:
S

son9701

Anh hok thấy là em chế từ cấy bài em hỏi anh à Đây nỳ.

Cho 3 số thực dương thỏa mãn [TEX]3abc \geq ab+bc+ca[/TEX] CMR:
[TEX]\frac{1}{\sqrt[]{a}}+\frac{1}{\sqrt[]{b}}+\frac{1}{\sqrt[]{c}} \geq \frac{9}{a^2+b^2+c^2}[/TEX]


P/s: Em del mấy cây spam đi nhá
[/B]


Giả thiết tg đg vs:
[TEX]\frac{1}{a}+\frac{1}{b}+\frac{1}{c} \leq 3[/TEX]
Mà [TEX]\frac{1}{a}+\frac{1}{b}+\frac{1}{c} \geq \frac{9}{a+b+c}[/TEX]
Nên [TEX]a+b+c \geq 3[/TEX]

Phần còn lại giải tg tự như vy000

ĐÃ BẢO LÀM SAI ĐỀ CÒN LÀM TƯƠNG TỰ CẤY CHI
 
Last edited by a moderator:
B

bosjeunhan


Cho 3 số thực dương thỏa mãn [TEX]3abc \geq ab+bc+ca[/TEX] CMR:
[TEX]\frac{1}{\sqrt[]{a}}+\frac{1}{\sqrt[]{b}}+\frac{1}{\sqrt[]{c}} \geq \frac{9}{a^2+b^2+c^2}[/TEX]


Đây ak, em chưa bài này, sơ lược thôi, lâu rồi mới có thời gian cho pic :x
Từ giả thiết, ta có:
[TEX]3 \geq \frac{1}{a}+\frac{1}{b}+\frac{1}{c}[/TEX]
Đặt $x=\frac{1}{a}$,...
Ta có: 3 \geq x+y+z
Áp dụng BĐT cauchy
[TEX]x^2+\sqrt[]{x}+\sqrt[]{x} \geq 3x [/TEX]
Tương tự, ta có:
[TEX]x^2+y^2+z^2+2.(\sqrt[]{x}+\sqrt[]{y}+\sqrt[]{z}) \geq 3.(x+y+z) \geq x^2+y^2+z^2+2.(xy+yz+xz)[/TEX]
[TEX]\sqrt[]{x}+\sqrt[]{y}+\sqrt[]{z} \geq xy+yz+xz[/TEX]
Đến đây thì xong rồi nhỉ
 
Last edited by a moderator:
B

bboy114crew


Đây ak, em chưa bài này, sơ lược thôi, lâu rồi mới có thời gian cho pic :x
Từ giả thiết, ta có:
[TEX]3 \geq \frac{1}{a}+\frac{1}{b}+\frac{1}{c}[/TEX]
Đặt $x=\frac{1}{a}$,...
Ta có:3\geq x+y+z
Áp dụng BĐT cauchy
[TEX]x^2+\sqrt[]{x}+\sqrt[]{x} \geq 3x [/TEX]
Tương tự, ta có:
[TEX]x^2+y^2+z^2+2.(\sqrt[]{x}+\sqrt[]{y}+\sqrt[]{z}) \geq 3.(x+y+z) \geq x^2+y^2+z^2+2.(xy+yz+xz)[/TEX]
[TEX]\sqrt[]{x}+\sqrt[]{y}+\sqrt[]{z} \geq xy+yz+xz[/TEX]
Đến đây thì xong rồi nhỉ
Anh chưa thấy có gì liên quan đến bài toán của em cả????

Em tưởng anh nhận ra rồi chứ ạ, vậy để em giải tiếp
 
Last edited by a moderator:
B

bosjeunhan


Đây ak, em chưa bài này, sơ lược thôi, lâu rồi mới có thời gian cho pic :x
Từ giả thiết, ta có:
[TEX]3\geq \frac{1}{a}+\frac{1}{b}+\frac{1}{c} [/TEX]
Đặt $x=\frac{1}{a}$,...
Ta có:3\geq x+y+z
Áp dụng BĐT cauchy
[TEX]x^2+\sqrt[]{x}+\sqrt[]{x} \geq 3x [/TEX]
Tương tự, ta có:
[TEX]x^2+y^2+z^2+2.(\sqrt[]{x}+\sqrt[]{y}+\sqrt[]{z}) \geq 3.(x+y+z) \geq x^2+y^2+z^2+2.(xy+yz+xz)[/TEX]
[TEX]\sqrt[]{x}+\sqrt[]{y}+\sqrt[]{z} \geq xy+yz+xz[/TEX]
Đến đây thì xong rồi nhỉ

Tiếp nhá ạ:
Suy ra được
[TEX]\frac{1}{\sqrt[]{a}}+\frac{1}{\sqrt[]{b}}+\frac{1}{\sqrt[]{c}} \geq \frac{1}{ab}+\frac{1}{bc}+\frac{1}{ca} \geq \frac{9}{ab+bc+ca} \geq \frac{9}{a^2+b^2+c^2}[/TEX]
 
Last edited by a moderator:
B

bo_ieu_tho

Thư giãn nào mấy bác, cho không khí đỡ căng thẳng :D
Bài 21: CM BĐT với u,y,e,ê > 0 và a,n,h \geq 1
$\frac{3.[(e-m)^2-(e+m)^2].[(y-1)^2-(y+1)^2]}{16.(a^3+n^3+h^3)}.\frac{ê}{u^{-1}} $ \leq :khi (79):

Chữa luôn nhể:

Áp dụng BĐT cauchy+biến đổi đại số, ta đưa về đc

$\frac{e.m.y.ê.u}{a.n.h}$ \leq :khi (79):
\Leftrightarrow $\frac{e.m.y.ê.u}{a.n.h}$ \leq $e.m.y.ê.u.a.n.h$ =))
\Leftrightarrow $(a.n.h)^2$ \geq 1


Luôn đúng
 
Last edited by a moderator:
R

rinnegan_97

mạo muội gửi 1 bài chế:

Bài 28: Cho a,b,c dương:

CMR: [TEX]\frac{a^3}{a^2+ab+b^2}+\frac{b^3}{b^2+bc+c^2}+ \frac{c^3}{c^2+ac+a^2} \geq\frac{1}{3}[/TEX]

bik [TEX]a^2+b^2+c^2=2[/TEX]
 
Last edited by a moderator:
B

bat.nap.quan.tai.hon.em.lan.cuoi

mạo muội gửi 1 bài chế:
cho a,b,c dương:

CMR: [TEX]\frac{a^3}{a^2+ab+b^2}+\frac{b^3}{b^2+bc+c^2}+ \frac{c^3}{c^2+ac+a^2} \geq\frac{1}{3}[/TEX]

bik [TEX]a^2+b^2+c^2=2[/TEX]

[TEX]\frac{a^3}{a^2+ab+b^2}+\frac{b^3}{b^2+bc+c^2}+ \frac{c^3}{c^2+ac+a^2}\geq \frac{(\sum a^2)^2}{\sum a^3+\sum_{sym} a^2b}=\frac{4}{\sum a. \sum a^2}\geq...[/TEX]

mà kiểu j lại ra 1/3 nhỉ :-/
 
H

hthtb22

Xin lỗi mọi người ,đính chính
Bài 29:
Cho a, b, c dương
Max{a,b,c} \leq 0.99
0 \leq x,y,z \leq 0.5;x+y+z=1
Tìm min
A(x,y,z)=ax+by+cz
 
Last edited by a moderator:
B

bat.nap.quan.tai.hon.em.lan.cuoi

Xin lỗi mọi người ,đính chính
Cho a, b, c dương
Max{a,b,c} \leq 0.99
0 \leq x,y,z \leq 0.5;x+y+z=1
Tìm min
A(x,y,z)=ax+by+cz

gs a=max{a,b,c}

A(x,y,z)=a(1-y-z)+by+cz

\\\\\\\\\=a+(b-a)y+(c-a)z

\\\\\\\\ \geq a+0.5(b-a)+0.5(c-a)

\\\\\\\\\=0.5(b+c)

:confused:

>>>>>>> [TEX]a,b,c \in (0; 0.99][/TEX]
 
Last edited by a moderator:
B

bosjeunhan


Bài 30: Cho $x,y,z>1$ và thỏa mãn $x^2+y^2+z^2=5$.
Chứng minh rằng: \[\frac{{{x}^{2}}}{y}+\frac{{{y}^{2}}}{z}+\frac{{{z}^{2}}}{x}\ge \frac{4}{\sqrt[]{7}}+\frac{1}{x}+\frac{1}{y}+\frac{1}{z}\]
 
Last edited by a moderator:
M

minhtuyb

Đề nghị bò đưa ra lời giải của bài trên.
---
Bài này là bài kiến nghị chung kết HMEO nhưng bị ông Sơn bác bỏ T.T (Lí do thương mem? @_@). Thôi cứ đưa lên đây ai chém được thì chém:



Bài 31: Cho $x,y,z$ thực thỏa mãn $x^2+y^2+z^2=1$. Tìm GTLN của:

$$xy(x^2+y^2+xy)+yz(y^2+y^2+yz)+zx(z^2+x^2+zx)$$

Tui cũng chế câu hình, thương mem quá, cụng bỏ đi roài =))
 
Last edited by a moderator:
S

son9701

Thêm 1 bài nữa nhìn khủng bố cho m.n mần:

Bài 32: Chứng minh rằng với mọi a;b;c là 3 cạnh 1 tam giác thì ta có:

$$(a^2+b^2+c^2)(a+b+c)(a+b-c)(b+c-a)(c+a-b) \le 9a^2b^2c^2$$
 
Last edited by a moderator:
Top Bottom